¿Cómo es posible cambiar la dirección de un giro mediante impulso?

En Weinberg QFT sección 2.5.5, define los estados de impulso pag por

Ψ pag , σ = tu ( L ( pag ) ) Ψ k , σ

hasta alguna normalización irrelevante, y L ( pag ) es la transformación de Lorentz que nos lleva de k m = ( metro , 0 , 0 , 0 ) a cualquier pag m con pag 2 = metro 2 y pag 0 > 0 .

En retrospectiva sé que σ etiqueta el giro de la partícula a lo largo del eje z, y que L ( pag ) es un impulso Entonces, según la definición anterior, este impulso no debería cambiar nuestro giro observado a lo largo del eje z.

Tengo j 3 Ψ k , σ = σ Ψ k , σ , así que por definición debería haber

j 3 [ tu ( L ( pag ) ) Ψ k , σ ] = σ [ tu ( L ( pag ) ) Ψ k , σ ]
Pero esto es inconsistente con el álgebra de Poincaré. De hecho, si solo impulso a lo largo de la z -eje entonces todo está bien, ya que j 3 y k 3 , el generador de impulsos a lo largo de la z -eje, conmutar. Pero no puedo llegar a lo arbitrario pag m solo impulsando a lo largo de la z -eje. solo puedo llegar a pag m = ( pag 0 , 0 , 0 , pag 3 ) .

No puedo esperar arreglar esto rotando alrededor z -eje tampoco, debo rotar alrededor del X o y eje. Pero estos tampoco viajan con j 3 , así que todavía tengo un problema.

¿Cómo puedo llegar a estados de arbitrariedad? pag m ?

Como ejemplo, supongamos que empiezo con un estado de giro definido en el marco de reposo, por lo que tengo

j 3 Ψ k , σ = σ Ψ k , σ
Ahora realizo un impulso infinitesimal en el y dirección para llegar a un nuevo estado
mi i k 2 pag 2 Ψ k , σ = ( 1 + i k 2 pag 2 ) Ψ k , σ
Ahora mido mi giro. Encuentro
j 3 ( 1 + i k 2 pag 2 ) Ψ k , σ = σ ( 1 + i k 2 pag 2 ) Ψ k , σ i k 1 pag 2 Ψ k , σ
El primer término es lo que quiero pero el segundo término arruina todo. ¿Cómo puedo resolver este problema?

¿Por qué deberías pensar que el operador de giro j 3 viaja con tu ( L , pag ) ? Por definición, el operador de espín es el componente del generador de grupos de Lorentz antisimétrico METRO ^ m v , y por lo tanto se transforma como el componente de tensor.
@NombreYYY Porque σ índice no cambia bajo la transformación, por lo que el estado transformado también debe ser un j 3 estado propio con el mismo valor propio σ . Como en el ejemplo que di, este no puede ser el caso a menos que j 3 viaja con tu ( L ( pag ) ) .
"...Debido a que el índice σ no cambia con la transformación, el estado transformado también debe ser un estado propio J3 con el mismo valor propio σ..." Es el número, pero el estado propio correspondiente está etiquetado por σ :
j 3 | k , σ = σ
Desde el pensamiento general la transformación de Lorentz tu ( Λ ) que actúa sobre | pag , σ lo convierte en la superposición de estados | Λ pag , σ con pesos de matriz C σ σ . Esto nuevamente muestra que su declaración es incorrecta.
No comprendo. Estoy de acuerdo en que una transformación general de Lorentz producirá una superposición de estados con diferentes σ con pesas C σ σ . Pero la transformación tu ( L ( pag ) ) es definido por Weinberg para no cambiar el σ índice, luego continúa diciendo que es específicamente un impulso, e incluso da una representación matricial específica para él.
Relacionado (¿posible duplicado?): physics.stackexchange.com/questions/240746/…

Respuestas (1)

Parece que tienes razón, y yo me equivoqué en la sección de comentarios. La respuesta a tu pregunta es simple: σ no significa la polarización para momentos arbitrarios pag m , aunque coincide con la polarización en reposo. Entonces, ¿cómo obtener la interpretación de la σ ¿etiqueta?

Por definición,

(1) tu ( L ( pag ) ) | k , σ = | pag , σ ,
entonces σ La cantidad de estado de una partícula, definida en reposo, no cambia bajo el impulso de Lorentz estándar. El significado físico de σ pues los momentos arbitrarios no son simples. En general, no coincide con el giro. j ^ 3 valor propio

Date cuenta primero que el grupo Little de órbita masiva del grupo de Lorentz es isomorfo a S O ( 3 ) grupo, cuyos generadores de representaciones irreductibles son j ^ i σ σ con σ = s , . . . , s ). Para que parezca que σ es simplemente el valor propio de j ^ 3 . Pero ahí está el problema.

Lo sabemos j ^ 3 se cambia bajo el impulso general de Lorentz como el componente del tensor antisimétrico; en el resultado llegamos a la afirmación de que σ se define como el valor propio de algún operador, cuya acción en el estado de una partícula con k m = ( metro , 0 ) coincide con la acción de j ^ 3 . Por el impulso general

(2) PAG m = ( mi , PAG ) , PAG 2 = metro 2 , PAG m = L m   v k v = ( L k ) m ,
sin embargo, esta correspondencia no es cierta, es decir, σ no es el valor propio de j ^ 3 y por lo tanto para momentos generales no es el giro.

Tal vez el operador que define el σ la etiqueta es el operador Pauli-Lubanski W ^ m multiplicado en L 1 ( pag ) :

(3) V ^ m L m   v ( pag ) W ^ v = 1 2 metro L m   v ( pag ) ϵ v α β γ pag ^ α j ^ β γ
Realmente, tenga en cuenta que debido a la invariancia de ϵ tensor tenemos que
L m   v ϵ v α β γ = ϵ m ρ d ϵ L   α ρ L   β d L   γ ϵ ,
de modo que, usando la relación
( L 1 ( pag ) pag ) m = metro d m 0
tenemos eso
V ^ m = { 0 , m = 0 1 2 ϵ 0 α β yo L α   d L β   k j ^ d k = j ^ yo , m = yo
Por lo tanto, tiene eso independientemente del valor del impulso general de 4 pag operador V ^ 3 , actuando sobre el estado | pag , σ , siempre dará σ valor como valor propio de j ^ 3 σ σ = σ d σ σ , con σ = s , . . . , s .

Me temo que todavía no estoy convencido. Al comienzo del capítulo 3 Weinberg dice; "Para etiquetar los estados de una partícula usamos sus cuatro momentos pag m , componente spin-z σ ." No hay nada que haga alusión a que este espín deba medirse en el marco de reposo, de hecho parece que se mide en el marco donde la partícula tiene momentos. pag m , a menos que esté malinterpretando completamente la oración.
@qftey: puede interpretar V ^ m = ( 0 , j ^ ) como la definición de la cantidad j ^ . Puede invertir la Ec. ( 3 ) para obtener la expresión del giro:
j ^ i = 1 2 ϵ i j k ( L 1 ( pag ) )   m j ( L 1 ( pag ) )   v k j ^ m v
Por lo tanto, la etiqueta de giro σ se interpreta como el momento angular de la partícula con momento pag , medido en el marco en el que esta partícula está en reposo (la transformación correspondiente viene dada por L 1 ( pag ) ).
No hay otra posibilidad si σ no cambia bajo impulsos.